Cracking The SAT Premium

(Marvins-Underground-K-12) #1

PRACTICE TEST 2 EXPLANATIONS


Section 1: Reading

1. A Although   Jane     resists    Mr.  Rochester,  the    first    sentence   of   the     passage    indicates    that    her
conscience and emotions are actually favorably inclined toward Mr. Rochester. Because
she has positive feelings toward him, (A) is an accurate description of her attitude. While
Mr. Rochester perceives Jane to be uncaring, (B) is incorrect because the narration
indicates that she does care but resists her own feelings. Similarly, (C) is incorrect because
the first sentence tells us she has positive feelings for Rochester. Because Jane acts calmly,
ignoring her emotions, she cannot accurately be described as reckless. Therefore, (D) is
also incorrect, and (A) is the correct answer.

2. B Because    questions   2   and 3   are general paired  questions,  consider    question    2   and the textual
evidence given in question 3 at the same time. Question 2 asks for the reason Jane refused
Rochester’s advances. Consider the lines referenced in question 3’s answer choices and
whether they support any of the answers in question 2. Choices (A), (C), and (D) of
question 3 do not support any of the answer choices in question 2, so they can be
eliminated. Choice (B) of question 3 (I will hold to the principles received by me when I
was sane, and not mad—as I am now) provides support only for (B) in question 2, so (B)
must be the answer to both question 2 and question 3. These answers make sense because
they indicate that Jane resists Rochester because she wishes to hold to her principles.

3. B See    the explanation for question    2.  As  noted   above,  (B) of  question    3   (I  will    hold    to  the
principles received by me when I was sane, and not mad—as I am now) provides support
only for (B) in question 2, so (B) must be the answer to both question 2 and question 3.
These answers make sense because they indicate that Jane resists Rochester because she
wishes to hold to her principles.

4. D There  is  no  evidence    that    Jane    is  literally   insane. Rather, she is  speaking    metaphorically
about the conflict between her reason and her desires. Therefore, (A) is wrong. Because
Jane says the law (not her mental state) has been given by God, (B) is incorrect. Jane says
that her insanity tempts her to disregard the worth of her principles, and it is her principles
that are preventing her from giving in to Mr. Rochester, so her insanity is tempting her to
give in to Rochester. Therefore, (C) is wrong; her insanity does not urge her to reject
Rochester, but rather to accept his advances. Choice (D) is correct because her feeling of
insanity is directly related to her emotions, as evidenced by the phrase with my veins
running fire, and my heart beating faster than I can count its throbs, which Jane provides
to explain why she feels insane.

5. C Because    wrought describes   Mr. Rochester’s fury,   and the passage says    that    his fury    has
reached its highest, wrought must mean something like “increased.” Because hammered has
nothing to do with increasing fury, (A) is incorrect. Choice (B) also does not have anything
to do with “increased,” so eliminate it. Because excited could mean “increased” when
Free download pdf